Proving if f'=g', then f=g+k for all k constant

  • Thread starter k3k3
  • Start date
  • Tags
    Constant
In summary: Suppose h(a)≠h(b)Then there is no point c in (a,b) so that h'(c)=0Things start going wrong about here. That is just plain not true. You are negating statements wrong, I think. What does h(a)≠h(b) really...do you know what "and" means?Right, sorry. I think I understand now.Let's say, for contradiction, that h(a)≠h(b). Then there exists some point c in (a,b) such that h'(c)=0.But since h'(x)=f'(x)-g'(x) and f'(x)=g'(
  • #1
k3k3
78
0

Homework Statement


Let f and g be two continuous functions that are defined on [a,b] and differentiable on (a,b).

Claim: If f'=g' on (a,b), then there exists a constant k such that f(x)=g(x)+k for all x in [a,b]


Homework Equations





The Attempt at a Solution



Let h:[a,b]->R be defined by h(x)=g(x)+k-f(x)

h is continuous and differentiable on [a,b] by the assumptions of f and g already made.

Since f'=g' => g'-f'=0

So h'(x) = g'(x) - f'(x) = 0

By the mean value theorem, there exists a c in (a,b) so that

0=(h(b)-h(a))/(b-a) => h(a)=h(b)

I think I showed rolle's theorem. I am trying to use my assumption that f'=g' to show the claim. Any hints on this?
 
Physics news on Phys.org
  • #2
Consider the function defined by [itex]h(x) = f(x)-g(x)[/itex] and apply the mean value theorem to show that this is a constant function.
 
  • #3
jgens said:
Consider the function defined by [itex]h(x) = f(x)-g(x)[/itex] and apply the mean value theorem to show that this is a constant function.

If h(x)=f(x)-g(x), then h'(x)=f'(x)-g'(x) and, since f'=g' by assumption, f'(x)-g'(x)=0 for all x in (a,b). By MVT, there is a c in (a,b) so that h'(c)=h(b)-h(a) = 0 or

[f(b)-g(b)-f(a)+g(a)]/(b-a) = 0 => f(b)-f(a)=g(b)-g(a) ?

If h'(x) = 0 for all x in (a,b), then h is constant.

Is this what you meant?
 
  • #4
k3k3 said:
Is this what you meant?

Not exactly. The idea is to suppose [itex]x,y \in (a,b)[/itex] and compute [itex]h(x)-h(y)[/itex] using the mean value theorem. This will tell you that the function is constant and complete the proof.
 
  • #5
jgens said:
Not exactly. The idea is to suppose [itex]x,y \in (a,b)[/itex] and compute [itex]h(x)-h(y)[/itex] using the mean value theorem. This will tell you that the function is constant and complete the proof.

So choose x,y in (a,b) arbitrary so that h(x)-h(y) = 0?
 
  • #6
k3k3 said:
So choose x,y in (a,b) arbitrary so that h(x)-h(y) = 0?

Do a proof by contradiction. Define h(x)=f(x)-g(x) and suppose it's not constant. What does the MVT tell you?
 
  • #7
k3k3 said:
So choose x,y in (a,b) arbitrary so that h(x)-h(y) = 0?

Yes, but you need to prove that, not just write it down.

RGV
 
  • #8
Dick said:
Do a proof by contradiction. Define h(x)=f(x)-g(x) and suppose it's not constant. What does the MVT tell you?

MVT tells me that there is a point in the interval (a,b) such that h'(c) = [h(b)-h(a)]/(b-a)

So f'(c)-g'(c) = [h(b)-h(a)]/(b-a) and f'(x) = g'(x) for all x in [a,b]

So 0 = [h(b)-h(a)]/(b-a) => h(b)=h(a) making it a constant function.
 
  • #9
k3k3 said:
MVT tells me that there is a point in the interval (a,b) such that h'(c) = [h(b)-h(a)]/(b-a)

So f'(c)-g'(c) = [h(b)-h(a)]/(b-a) and f'(x) = g'(x) for all x in [a,b]

So 0 = [h(b)-h(a)]/(b-a) => h(b)=h(a) making it a constant function.

You aren't really stating that it in "proof by contradiction" form. Suppose h(a) is NOT equal to h(b) what would bother you about that?
 
  • #10
Dick said:
You aren't really stating that it in "proof by contradiction" form. Suppose h(a) is NOT equal to h(b) what would bother you about that?

What would bother me that is, when f'(x)=g'(x), is that h(a) is h(b). And when h'(x) = 0 for all x in (a,b), then h(x) is constant.
 
  • #11
k3k3 said:
What would bother me that is, when f'(x)=g'(x), is that h(a) is h(b). And when h'(x) = 0 for all x in (a,b), then h(x) is constant.

You are just stating what you already think you know without proving anything. If h(a)-h(b) is not zero then (h(a)-h(b))/(a-b) is not zero. The MVT should give you a contradiction that would prove that can't happen.
 
  • #12
Dick said:
You are just stating what you already think you know without proving anything. If h(a)-h(b) is not zero then (h(a)-h(b))/(a-b) is not zero. The MVT should give you a contradiction that would prove that can't happen.

Let me try this again, then...

I want to show that if f'=g' on (a,b), then there exists a constant k such that f(x)=g(x)+k for all x in [a,b]

Let f and g be two continuous functions that are defined on [a,b] and differentiable on (a,b).
Assume f'(x)=g'(x) for all x in [a,b]

Let h:[a,b]->ℝ be defined by h(x)=f(x)-g(x)

Suppose h(a)≠h(b)

Then there is no point c in (a,b) so that h'(c)=0

By the MVT, there is a point d in (x,y)[itex]\subset[/itex][a,b] such that h'(d) = [h(y)-h(x)]/(y-x)

Since h is differentiable on all of [a,b], h'(x)=f'(x)-g'(x)

And f'-g'=0 by assumption.

So h'(d)=0=[h(y)-h(x)]/(y-x) => h(y) = h(x) for all x,y in [a,b]

This means h'(x)=0 for all x in [a,b] and by the monotonicity theorem, h is constant on [a,b].

So f and g must be the same.
 
  • #13
k3k3 said:
Suppose h(a)≠h(b)

Then there is no point c in (a,b) so that h'(c)=0

Things start going wrong about here. That is just plain not true. You are negating statements wrong, I think. What does h(a)≠h(b) really imply?
 
  • #14
Dick said:
Things start going wrong about here. That is just plain not true. You are negating statements wrong, I think. What does h(a)≠h(b) really imply?

That the function has different values at the end points of the interval.
 
  • #15
k3k3 said:
That the function has different values at the end points of the interval.

What does h(a)≠h(b) imply if you use the mean value theorem?
 
  • #16
Dick said:
What does h(a)≠h(b) imply if you use the mean value theorem?

h'(c) = [h(b)-h(a)]/(b-a) ≠ 0
 
  • #17
k3k3 said:
h'(c) = [h(b)-h(a)]/(b-a) ≠ 0

Yes! There is c in (a,b) such that h'(c)≠0. But you know h'(x)=0 for all points c in (a,b). So?
 
  • #18
Dick said:
Yes! There is c in (a,b) such that h'(c)≠0. But you know h'(x)=0 for all points c in (a,b). So?

h'(c) must be 0 too since c is in (a,b)
 
  • #19
k3k3 said:
h'(c) must be 0 too since c is in (a,b)

Yes. So h'(c) must be both zero and nonzero. That's impossible, so your initial assumption h(a)≠h(b) must be wrong. So?
 

1. How do you prove if f'=g', then f=g+k for all k constant?

To prove this statement, we can use the Mean Value Theorem. This theorem states that if a function f is continuous on the closed interval [a,b] and differentiable on the open interval (a,b), then there exists a point c in (a,b) such that f'(c) = (f(b) - f(a))/(b-a). We can use this theorem to show that f and g differ by a constant k.

2. What is the significance of g' and f' being equal in this statement?

The equality of g' and f' means that the slopes of the two functions are equal at every point. This is a strong indication that the two functions are closely related and may differ by a constant.

3. Can you provide an example to demonstrate this statement?

Sure, let's take the functions f(x) = x^2 and g(x) = x^2 + 3. Both of these functions have the same derivative, which is 2x. This means that f' and g' are equal. Using the Mean Value Theorem, we can show that f and g differ by a constant k=3, as (f(1) - f(0))/(1-0) = (1^2 - 0^2)/1 = 1 and (g(1) - g(0))/(1-0) = (1^2 + 3 - 0^2)/1 = 4, and 4-1=3.

4. What if f and g are not continuous or differentiable on the same interval?

If f and g are not continuous on the same interval, then the statement may not hold true. For example, if f(x) = 1/x and g(x) = 1/x^2, then f' and g' are equal, but f and g are not equal on any interval. Similarly, if f and g are not differentiable on the same interval, then we cannot use the Mean Value Theorem to prove the statement.

5. Can this statement be extended to non-constant k values?

Yes, the statement can be extended to any k value, not just constant ones. This is because if f and g differ by a constant k, then their derivatives will also differ by a constant k. Therefore, if f'=g', then f and g must differ by a constant, regardless of its value.

Similar threads

  • Calculus and Beyond Homework Help
Replies
2
Views
160
  • Calculus and Beyond Homework Help
Replies
2
Views
1K
  • Calculus and Beyond Homework Help
Replies
1
Views
284
  • Calculus and Beyond Homework Help
Replies
8
Views
474
  • Calculus and Beyond Homework Help
Replies
5
Views
1K
  • Calculus and Beyond Homework Help
Replies
2
Views
843
  • Calculus and Beyond Homework Help
Replies
14
Views
1K
  • Calculus and Beyond Homework Help
Replies
6
Views
562
  • Calculus and Beyond Homework Help
Replies
2
Views
1K
  • Calculus and Beyond Homework Help
Replies
10
Views
1K
Back
Top